2

I am going over old exam and am not understanding the logic behind the answer given in the mark-scheme.

A beam of protons and antiprotons attain energies of 1400 GeV in a synchrotron. Why is it that, even at the very high energies of this particle beam, the collisions are unlikely to lead to the creation of a single unbound quark.

The mark-scheme, very confusingly, answer this as follows:

as quarks separate the force between them is constant;

(so) energy required to release single quark increases with distance;

these energies exceed the threshold for meson (pair) production;

Each part, separated by a semicolon, gives one point (so that you understand why its so oddly formatted).

I get that we should simply know that the energy transferred between the quarks gets bigger with distance (work = force * distance), and that force is constant, but why does it jump to the conclusion that these energies exceed what we already have? And why is the meson (pair) production relevant?

Surculus
  • 121
  • This is the usual hand-wavy explanation of "confinement". If you are not familiar with that term, it's underlying logic and it's implication then this is something that you should probably bone up on. – dmckee --- ex-moderator kitten Dec 10 '13 at 20:19
  • @dmckee: I am familiar with the concept, and at first I simply wrote "Quarks cannot exist outside of the confinement of another particle," and then scratched my head at why this was three points. So it is basically that because it cannot happen? The third line is a very weird way to express this, I think. – Surculus Dec 10 '13 at 20:46
  • 1
    Just knowing that quarks are confined is only half the battle. Go read up on this. That rubric is expressing a very conventional picture of the process. Or there are various posts about it around the site. I have one on http://physics.stackexchange.com/questions/18952/hadron-jets-from-quark-antiquark-in-colliders. – dmckee --- ex-moderator kitten Dec 10 '13 at 21:04

0 Answers0